A Better Estimate of A Big Number!

Algebra Level pending

Let A A , B B and N N be real numbers. We say that A A is a better estimate of N N than B B if N A < N B |N - A| < |N - B| is fulfilled.

Which of the following is the best estimate of 68 9 1679 689^{1679} ?


Side-note: both 689 and 1679 have special meanings in the minds of Hong Kong people.
9 4995 9^{4995} 9 4997 9^{4997} 9 4996 9^{4996} 9 4900 9^{4900}

This section requires Javascript.
You are seeing this because something didn't load right. We suggest you, (a) try refreshing the page, (b) enabling javascript if it is disabled on your browser and, finally, (c) loading the non-javascript version of this page . We're sorry about the hassle.

1 solution

Wing Tang
Feb 9, 2016

Solution: \textbf{Solution:}

Let a a be a real number such that 9 a = 68 9 1679 9^a = 689^{1679} . Taking log \log on both sides yields

a = 1679 log 689 log 9 4993.877 a = \frac{1679 \log 689}{\log 9} \approx 4993.877 .

Clearly, 4900 < a < 4995 4900 < a < 4995 , for which we can eliminate two choices, 9 4996 9^{4996} and 9 4997 9^{4997} . Now we only compare 9 4900 9^{4900} and 9 4995 9^{4995} .

9 4995 9 a 9 a 9 4900 = 9 95 9 a 4900 9 a 4900 1 > 9 95 9 a 4900 9 a 4900 = 9 4995 a 1 > 1 \displaystyle \frac{9^{4995} - 9^a}{9^a - 9^{4900}} = \frac{9^{95} - 9^{a-4900}}{9^{a-4900} - 1} > \frac{9^{95} - 9^{a-4900}}{9^{a-4900}} = 9^{4995 - a} -1 > 1

Now, 9 4995 68 9 1679 > 68 9 1679 9 4900 |9^{4995} - 689^{1679}| > |689^{1679} - 9^{4900}| and so the best estimate is 9 4900 9^{4900} . []

\\ \\

Alternative Solution \textbf{Alternative Solution} :

Let a a be a real number such that 9 a = 68 9 1679 9^a = 689^{1679} . Taking log \log on both sides yields

a = 1679 log 689 log 9 4993.877 a = \frac{1679 \log 689}{\log 9} \approx 4993.877 .

Clearly, 4900 < a < 4995 4900 < a < 4995 . So 9 4900 < 9 a < 9 4995 9^{4900} < 9^a < 9^{4995} , for which we can eliminate two choices 9 4996 9^{4996} and 9 4997 9^{4997} . Now we consider the arithmetic mean of 9 4995 9^{4995} and 9 4900 9^{4900} :

log 9 ( 9 4900 + 9 4995 2 ) = log 9 ( 9 4900 ( 1 + 9 95 ) 2 ) = log 9 9 4900 + log 9 ( 1 + 9 95 ) log 9 2 > 4900 + log 9 9 95 log 9 2 = 4995 log 2 log 9 4994.68 > a \begin{aligned} \displaystyle \log_9 \left(\frac{9^{4900} + 9^{4995}}{2}\right) &=\log_9 \left(\frac{9^{4900}\left(1 + 9^{95}\right)}{2}\right)\\ &= \log_9 {9^{4900}} + \log_9 \left(1 + 9^{95}\right) - \log_9 2\\ & > 4900 + \log_9 9^{95} - \log_9 2\\ &= 4995 - \frac{\log 2}{\log 9}\\ & \approx 4994.68 > a \end{aligned}

Hence

9 4900 < 9 a < 9 4900 + 9 4995 2 . \displaystyle 9^{4900} < 9^a < \frac{9^{4900} + 9^{4995}}{2}.

This means

9 4995 68 9 1679 > 68 9 1679 9 4900 |9^{4995} - 689^{1679}| > |689^{1679} - 9^{4900}| and so the best estimate is 9 4900 9^{4900} . []

0 pending reports

×

Problem Loading...

Note Loading...

Set Loading...